Do not understand L'Hopital Rule











up vote
2
down vote

favorite
2













I have just started learning L'Hopital rule, and so far I thought I understood everything until I stumbled upon this question
$$lim_{xto 0} frac{ln(cos(ax))}{ln(cos(bx))}.$$




To this, eventually got
$$lim_{xto 0} frac{a sin(ax) cos(bx)}{b sin(bx)cos(ax)}$$



From my knowledge, $sin(0)$ is 0!! and the whole thing will be ''$frac{0}{0}$'', however the answer key I was given does not continue the implementation of the L'Hopital rule, but instead obtains the answer $frac{a^2}{b^2}$.



Is there some important concept I'm missing out? Or is the differentiation supposed to continue and the answer key just skipped the steps?










share|cite|improve this question




















  • 2




    Short answer: since $sin(ax), sin(bx) xrightarrow{x to 0} 0$, you have the apply L'Hospital's rule a second time.
    – Viktor Glombik
    Nov 17 at 10:58










  • Are you asking why your answer key does not use L'Hopital rule although the limit is of the form $0/0$, or are you getting stuck while applying the rule?
    – астон вілла олоф мэллбэрг
    Nov 17 at 10:59






  • 1




    Note that $lim_{xto 0}frac{sin(ax)}{x}=a$
    – Robert Z
    Nov 17 at 11:00












  • @астонвіллаолофмэллбэрг the question does use the rule but the answer doesn’t tally up with what i know and i was just wondering if there are gaps in my knowledge, like the derivative i came up with is supposed to be 0/0 but the answer key shows a^2/b^2
    – Ranice Tan
    Nov 17 at 11:00






  • 1




    @RaniceTan For getting a better idea of how you and the answer key differ, reproduce the answer from the answer key here, and then show your own working and why you are getting $0/0$ where the answer key is getting $a^2/b^2$.
    – астон вілла олоф мэллбэрг
    Nov 17 at 11:03















up vote
2
down vote

favorite
2













I have just started learning L'Hopital rule, and so far I thought I understood everything until I stumbled upon this question
$$lim_{xto 0} frac{ln(cos(ax))}{ln(cos(bx))}.$$




To this, eventually got
$$lim_{xto 0} frac{a sin(ax) cos(bx)}{b sin(bx)cos(ax)}$$



From my knowledge, $sin(0)$ is 0!! and the whole thing will be ''$frac{0}{0}$'', however the answer key I was given does not continue the implementation of the L'Hopital rule, but instead obtains the answer $frac{a^2}{b^2}$.



Is there some important concept I'm missing out? Or is the differentiation supposed to continue and the answer key just skipped the steps?










share|cite|improve this question




















  • 2




    Short answer: since $sin(ax), sin(bx) xrightarrow{x to 0} 0$, you have the apply L'Hospital's rule a second time.
    – Viktor Glombik
    Nov 17 at 10:58










  • Are you asking why your answer key does not use L'Hopital rule although the limit is of the form $0/0$, or are you getting stuck while applying the rule?
    – астон вілла олоф мэллбэрг
    Nov 17 at 10:59






  • 1




    Note that $lim_{xto 0}frac{sin(ax)}{x}=a$
    – Robert Z
    Nov 17 at 11:00












  • @астонвіллаолофмэллбэрг the question does use the rule but the answer doesn’t tally up with what i know and i was just wondering if there are gaps in my knowledge, like the derivative i came up with is supposed to be 0/0 but the answer key shows a^2/b^2
    – Ranice Tan
    Nov 17 at 11:00






  • 1




    @RaniceTan For getting a better idea of how you and the answer key differ, reproduce the answer from the answer key here, and then show your own working and why you are getting $0/0$ where the answer key is getting $a^2/b^2$.
    – астон вілла олоф мэллбэрг
    Nov 17 at 11:03













up vote
2
down vote

favorite
2









up vote
2
down vote

favorite
2






2






I have just started learning L'Hopital rule, and so far I thought I understood everything until I stumbled upon this question
$$lim_{xto 0} frac{ln(cos(ax))}{ln(cos(bx))}.$$




To this, eventually got
$$lim_{xto 0} frac{a sin(ax) cos(bx)}{b sin(bx)cos(ax)}$$



From my knowledge, $sin(0)$ is 0!! and the whole thing will be ''$frac{0}{0}$'', however the answer key I was given does not continue the implementation of the L'Hopital rule, but instead obtains the answer $frac{a^2}{b^2}$.



Is there some important concept I'm missing out? Or is the differentiation supposed to continue and the answer key just skipped the steps?










share|cite|improve this question
















I have just started learning L'Hopital rule, and so far I thought I understood everything until I stumbled upon this question
$$lim_{xto 0} frac{ln(cos(ax))}{ln(cos(bx))}.$$




To this, eventually got
$$lim_{xto 0} frac{a sin(ax) cos(bx)}{b sin(bx)cos(ax)}$$



From my knowledge, $sin(0)$ is 0!! and the whole thing will be ''$frac{0}{0}$'', however the answer key I was given does not continue the implementation of the L'Hopital rule, but instead obtains the answer $frac{a^2}{b^2}$.



Is there some important concept I'm missing out? Or is the differentiation supposed to continue and the answer key just skipped the steps?







calculus real-analysis limits derivatives






share|cite|improve this question















share|cite|improve this question













share|cite|improve this question




share|cite|improve this question








edited Nov 17 at 11:26









Argon

204




204










asked Nov 17 at 10:55









Ranice Tan

182




182








  • 2




    Short answer: since $sin(ax), sin(bx) xrightarrow{x to 0} 0$, you have the apply L'Hospital's rule a second time.
    – Viktor Glombik
    Nov 17 at 10:58










  • Are you asking why your answer key does not use L'Hopital rule although the limit is of the form $0/0$, or are you getting stuck while applying the rule?
    – астон вілла олоф мэллбэрг
    Nov 17 at 10:59






  • 1




    Note that $lim_{xto 0}frac{sin(ax)}{x}=a$
    – Robert Z
    Nov 17 at 11:00












  • @астонвіллаолофмэллбэрг the question does use the rule but the answer doesn’t tally up with what i know and i was just wondering if there are gaps in my knowledge, like the derivative i came up with is supposed to be 0/0 but the answer key shows a^2/b^2
    – Ranice Tan
    Nov 17 at 11:00






  • 1




    @RaniceTan For getting a better idea of how you and the answer key differ, reproduce the answer from the answer key here, and then show your own working and why you are getting $0/0$ where the answer key is getting $a^2/b^2$.
    – астон вілла олоф мэллбэрг
    Nov 17 at 11:03














  • 2




    Short answer: since $sin(ax), sin(bx) xrightarrow{x to 0} 0$, you have the apply L'Hospital's rule a second time.
    – Viktor Glombik
    Nov 17 at 10:58










  • Are you asking why your answer key does not use L'Hopital rule although the limit is of the form $0/0$, or are you getting stuck while applying the rule?
    – астон вілла олоф мэллбэрг
    Nov 17 at 10:59






  • 1




    Note that $lim_{xto 0}frac{sin(ax)}{x}=a$
    – Robert Z
    Nov 17 at 11:00












  • @астонвіллаолофмэллбэрг the question does use the rule but the answer doesn’t tally up with what i know and i was just wondering if there are gaps in my knowledge, like the derivative i came up with is supposed to be 0/0 but the answer key shows a^2/b^2
    – Ranice Tan
    Nov 17 at 11:00






  • 1




    @RaniceTan For getting a better idea of how you and the answer key differ, reproduce the answer from the answer key here, and then show your own working and why you are getting $0/0$ where the answer key is getting $a^2/b^2$.
    – астон вілла олоф мэллбэрг
    Nov 17 at 11:03








2




2




Short answer: since $sin(ax), sin(bx) xrightarrow{x to 0} 0$, you have the apply L'Hospital's rule a second time.
– Viktor Glombik
Nov 17 at 10:58




Short answer: since $sin(ax), sin(bx) xrightarrow{x to 0} 0$, you have the apply L'Hospital's rule a second time.
– Viktor Glombik
Nov 17 at 10:58












Are you asking why your answer key does not use L'Hopital rule although the limit is of the form $0/0$, or are you getting stuck while applying the rule?
– астон вілла олоф мэллбэрг
Nov 17 at 10:59




Are you asking why your answer key does not use L'Hopital rule although the limit is of the form $0/0$, or are you getting stuck while applying the rule?
– астон вілла олоф мэллбэрг
Nov 17 at 10:59




1




1




Note that $lim_{xto 0}frac{sin(ax)}{x}=a$
– Robert Z
Nov 17 at 11:00






Note that $lim_{xto 0}frac{sin(ax)}{x}=a$
– Robert Z
Nov 17 at 11:00














@астонвіллаолофмэллбэрг the question does use the rule but the answer doesn’t tally up with what i know and i was just wondering if there are gaps in my knowledge, like the derivative i came up with is supposed to be 0/0 but the answer key shows a^2/b^2
– Ranice Tan
Nov 17 at 11:00




@астонвіллаолофмэллбэрг the question does use the rule but the answer doesn’t tally up with what i know and i was just wondering if there are gaps in my knowledge, like the derivative i came up with is supposed to be 0/0 but the answer key shows a^2/b^2
– Ranice Tan
Nov 17 at 11:00




1




1




@RaniceTan For getting a better idea of how you and the answer key differ, reproduce the answer from the answer key here, and then show your own working and why you are getting $0/0$ where the answer key is getting $a^2/b^2$.
– астон вілла олоф мэллбэрг
Nov 17 at 11:03




@RaniceTan For getting a better idea of how you and the answer key differ, reproduce the answer from the answer key here, and then show your own working and why you are getting $0/0$ where the answer key is getting $a^2/b^2$.
– астон вілла олоф мэллбэрг
Nov 17 at 11:03










4 Answers
4






active

oldest

votes

















up vote
1
down vote



accepted










You can apply l'Hôpital once more:
$$
lim_{xto 0}frac{a}{b}
frac{acos(ax)cos(bx)-bsin(ax)sin(bx)}
{bcos(bx)cos(ax)-asin(bx)sin(ax)}
$$

and now direct substitution is possible. Not the best method, however: you can observe that
$$
lim_{xto0}frac{cos(bx)}{cos(ax)}=1
$$

and reduce to computing
$$
lim_{xto0}frac{sin(ax)}{sin(bx)}
$$

which is $a/b$.



With Taylor expansion, note that, assuming $ane0$,
$$
cos(ax)=1-frac{(ax)^2}{2!}+o(x^2)qquad ln(1+t)=t+o(t)
$$

so that
$$
ln(cos(ax))=-frac{a^2x^2}{2}+o(x^2)
$$

Hence your limit is
$$
lim_{xto0}frac{-a^2x^2/2+o(x^2)}{-b^2x^2/2+o(x^2)}=frac{a^2}{b^2}
$$






share|cite|improve this answer




























    up vote
    1
    down vote













    $$lim_{xto}dfrac{sin ax}{sin bx}cdotdfrac{cos bx}{cos ax}$$



    $$=dfrac ablim_{xto0}dfrac{sin ax}{ax}cdotlim_{xto0}dfrac{bx}{sin bx}lim_{xto0}dfrac{cos bx}{cos ax}=?$$



    Mind you $lim_{xto0}implies xne0$






    share|cite|improve this answer




























      up vote
      1
      down vote













      Using $cos^2y=1-sin^2y,$



      $$2lim_{xto0}dfrac{ln(cos ax)}{x^2}=-a^2lim_{xto0}dfrac{ln(1-sin^2ax)}{-sin^2ax}left(lim_{xto0}dfrac{sin(ax)}{ax}right)^2=-a^2$$






      share|cite|improve this answer






























        up vote
        0
        down vote













        From here by standard limits



        $$ldots=lim_{xto 0} frac{a sin(ax) cos(bx)}{b sin(bx)cos(ax)}=lim_{xto 0} frac a bfrac{tan (ax)}{tan (bx)}=lim_{xto 0} frac {a^2} {b^2}frac{tan (ax)}{ax}frac{bx}{tan (bx)}=frac {a^2} {b^2}cdot 1 cdot 1 =frac {a^2} {b^2}$$






        share|cite|improve this answer





















          Your Answer





          StackExchange.ifUsing("editor", function () {
          return StackExchange.using("mathjaxEditing", function () {
          StackExchange.MarkdownEditor.creationCallbacks.add(function (editor, postfix) {
          StackExchange.mathjaxEditing.prepareWmdForMathJax(editor, postfix, [["$", "$"], ["\\(","\\)"]]);
          });
          });
          }, "mathjax-editing");

          StackExchange.ready(function() {
          var channelOptions = {
          tags: "".split(" "),
          id: "69"
          };
          initTagRenderer("".split(" "), "".split(" "), channelOptions);

          StackExchange.using("externalEditor", function() {
          // Have to fire editor after snippets, if snippets enabled
          if (StackExchange.settings.snippets.snippetsEnabled) {
          StackExchange.using("snippets", function() {
          createEditor();
          });
          }
          else {
          createEditor();
          }
          });

          function createEditor() {
          StackExchange.prepareEditor({
          heartbeatType: 'answer',
          convertImagesToLinks: true,
          noModals: true,
          showLowRepImageUploadWarning: true,
          reputationToPostImages: 10,
          bindNavPrevention: true,
          postfix: "",
          imageUploader: {
          brandingHtml: "Powered by u003ca class="icon-imgur-white" href="https://imgur.com/"u003eu003c/au003e",
          contentPolicyHtml: "User contributions licensed under u003ca href="https://creativecommons.org/licenses/by-sa/3.0/"u003ecc by-sa 3.0 with attribution requiredu003c/au003e u003ca href="https://stackoverflow.com/legal/content-policy"u003e(content policy)u003c/au003e",
          allowUrls: true
          },
          noCode: true, onDemand: true,
          discardSelector: ".discard-answer"
          ,immediatelyShowMarkdownHelp:true
          });


          }
          });














          draft saved

          draft discarded


















          StackExchange.ready(
          function () {
          StackExchange.openid.initPostLogin('.new-post-login', 'https%3a%2f%2fmath.stackexchange.com%2fquestions%2f3002207%2fdo-not-understand-lhopital-rule%23new-answer', 'question_page');
          }
          );

          Post as a guest















          Required, but never shown

























          4 Answers
          4






          active

          oldest

          votes








          4 Answers
          4






          active

          oldest

          votes









          active

          oldest

          votes






          active

          oldest

          votes








          up vote
          1
          down vote



          accepted










          You can apply l'Hôpital once more:
          $$
          lim_{xto 0}frac{a}{b}
          frac{acos(ax)cos(bx)-bsin(ax)sin(bx)}
          {bcos(bx)cos(ax)-asin(bx)sin(ax)}
          $$

          and now direct substitution is possible. Not the best method, however: you can observe that
          $$
          lim_{xto0}frac{cos(bx)}{cos(ax)}=1
          $$

          and reduce to computing
          $$
          lim_{xto0}frac{sin(ax)}{sin(bx)}
          $$

          which is $a/b$.



          With Taylor expansion, note that, assuming $ane0$,
          $$
          cos(ax)=1-frac{(ax)^2}{2!}+o(x^2)qquad ln(1+t)=t+o(t)
          $$

          so that
          $$
          ln(cos(ax))=-frac{a^2x^2}{2}+o(x^2)
          $$

          Hence your limit is
          $$
          lim_{xto0}frac{-a^2x^2/2+o(x^2)}{-b^2x^2/2+o(x^2)}=frac{a^2}{b^2}
          $$






          share|cite|improve this answer

























            up vote
            1
            down vote



            accepted










            You can apply l'Hôpital once more:
            $$
            lim_{xto 0}frac{a}{b}
            frac{acos(ax)cos(bx)-bsin(ax)sin(bx)}
            {bcos(bx)cos(ax)-asin(bx)sin(ax)}
            $$

            and now direct substitution is possible. Not the best method, however: you can observe that
            $$
            lim_{xto0}frac{cos(bx)}{cos(ax)}=1
            $$

            and reduce to computing
            $$
            lim_{xto0}frac{sin(ax)}{sin(bx)}
            $$

            which is $a/b$.



            With Taylor expansion, note that, assuming $ane0$,
            $$
            cos(ax)=1-frac{(ax)^2}{2!}+o(x^2)qquad ln(1+t)=t+o(t)
            $$

            so that
            $$
            ln(cos(ax))=-frac{a^2x^2}{2}+o(x^2)
            $$

            Hence your limit is
            $$
            lim_{xto0}frac{-a^2x^2/2+o(x^2)}{-b^2x^2/2+o(x^2)}=frac{a^2}{b^2}
            $$






            share|cite|improve this answer























              up vote
              1
              down vote



              accepted







              up vote
              1
              down vote



              accepted






              You can apply l'Hôpital once more:
              $$
              lim_{xto 0}frac{a}{b}
              frac{acos(ax)cos(bx)-bsin(ax)sin(bx)}
              {bcos(bx)cos(ax)-asin(bx)sin(ax)}
              $$

              and now direct substitution is possible. Not the best method, however: you can observe that
              $$
              lim_{xto0}frac{cos(bx)}{cos(ax)}=1
              $$

              and reduce to computing
              $$
              lim_{xto0}frac{sin(ax)}{sin(bx)}
              $$

              which is $a/b$.



              With Taylor expansion, note that, assuming $ane0$,
              $$
              cos(ax)=1-frac{(ax)^2}{2!}+o(x^2)qquad ln(1+t)=t+o(t)
              $$

              so that
              $$
              ln(cos(ax))=-frac{a^2x^2}{2}+o(x^2)
              $$

              Hence your limit is
              $$
              lim_{xto0}frac{-a^2x^2/2+o(x^2)}{-b^2x^2/2+o(x^2)}=frac{a^2}{b^2}
              $$






              share|cite|improve this answer












              You can apply l'Hôpital once more:
              $$
              lim_{xto 0}frac{a}{b}
              frac{acos(ax)cos(bx)-bsin(ax)sin(bx)}
              {bcos(bx)cos(ax)-asin(bx)sin(ax)}
              $$

              and now direct substitution is possible. Not the best method, however: you can observe that
              $$
              lim_{xto0}frac{cos(bx)}{cos(ax)}=1
              $$

              and reduce to computing
              $$
              lim_{xto0}frac{sin(ax)}{sin(bx)}
              $$

              which is $a/b$.



              With Taylor expansion, note that, assuming $ane0$,
              $$
              cos(ax)=1-frac{(ax)^2}{2!}+o(x^2)qquad ln(1+t)=t+o(t)
              $$

              so that
              $$
              ln(cos(ax))=-frac{a^2x^2}{2}+o(x^2)
              $$

              Hence your limit is
              $$
              lim_{xto0}frac{-a^2x^2/2+o(x^2)}{-b^2x^2/2+o(x^2)}=frac{a^2}{b^2}
              $$







              share|cite|improve this answer












              share|cite|improve this answer



              share|cite|improve this answer










              answered Nov 17 at 11:25









              egreg

              175k1383198




              175k1383198






















                  up vote
                  1
                  down vote













                  $$lim_{xto}dfrac{sin ax}{sin bx}cdotdfrac{cos bx}{cos ax}$$



                  $$=dfrac ablim_{xto0}dfrac{sin ax}{ax}cdotlim_{xto0}dfrac{bx}{sin bx}lim_{xto0}dfrac{cos bx}{cos ax}=?$$



                  Mind you $lim_{xto0}implies xne0$






                  share|cite|improve this answer

























                    up vote
                    1
                    down vote













                    $$lim_{xto}dfrac{sin ax}{sin bx}cdotdfrac{cos bx}{cos ax}$$



                    $$=dfrac ablim_{xto0}dfrac{sin ax}{ax}cdotlim_{xto0}dfrac{bx}{sin bx}lim_{xto0}dfrac{cos bx}{cos ax}=?$$



                    Mind you $lim_{xto0}implies xne0$






                    share|cite|improve this answer























                      up vote
                      1
                      down vote










                      up vote
                      1
                      down vote









                      $$lim_{xto}dfrac{sin ax}{sin bx}cdotdfrac{cos bx}{cos ax}$$



                      $$=dfrac ablim_{xto0}dfrac{sin ax}{ax}cdotlim_{xto0}dfrac{bx}{sin bx}lim_{xto0}dfrac{cos bx}{cos ax}=?$$



                      Mind you $lim_{xto0}implies xne0$






                      share|cite|improve this answer












                      $$lim_{xto}dfrac{sin ax}{sin bx}cdotdfrac{cos bx}{cos ax}$$



                      $$=dfrac ablim_{xto0}dfrac{sin ax}{ax}cdotlim_{xto0}dfrac{bx}{sin bx}lim_{xto0}dfrac{cos bx}{cos ax}=?$$



                      Mind you $lim_{xto0}implies xne0$







                      share|cite|improve this answer












                      share|cite|improve this answer



                      share|cite|improve this answer










                      answered Nov 17 at 11:01









                      lab bhattacharjee

                      221k15155272




                      221k15155272






















                          up vote
                          1
                          down vote













                          Using $cos^2y=1-sin^2y,$



                          $$2lim_{xto0}dfrac{ln(cos ax)}{x^2}=-a^2lim_{xto0}dfrac{ln(1-sin^2ax)}{-sin^2ax}left(lim_{xto0}dfrac{sin(ax)}{ax}right)^2=-a^2$$






                          share|cite|improve this answer



























                            up vote
                            1
                            down vote













                            Using $cos^2y=1-sin^2y,$



                            $$2lim_{xto0}dfrac{ln(cos ax)}{x^2}=-a^2lim_{xto0}dfrac{ln(1-sin^2ax)}{-sin^2ax}left(lim_{xto0}dfrac{sin(ax)}{ax}right)^2=-a^2$$






                            share|cite|improve this answer

























                              up vote
                              1
                              down vote










                              up vote
                              1
                              down vote









                              Using $cos^2y=1-sin^2y,$



                              $$2lim_{xto0}dfrac{ln(cos ax)}{x^2}=-a^2lim_{xto0}dfrac{ln(1-sin^2ax)}{-sin^2ax}left(lim_{xto0}dfrac{sin(ax)}{ax}right)^2=-a^2$$






                              share|cite|improve this answer














                              Using $cos^2y=1-sin^2y,$



                              $$2lim_{xto0}dfrac{ln(cos ax)}{x^2}=-a^2lim_{xto0}dfrac{ln(1-sin^2ax)}{-sin^2ax}left(lim_{xto0}dfrac{sin(ax)}{ax}right)^2=-a^2$$







                              share|cite|improve this answer














                              share|cite|improve this answer



                              share|cite|improve this answer








                              edited Nov 17 at 11:42

























                              answered Nov 17 at 11:32









                              lab bhattacharjee

                              221k15155272




                              221k15155272






















                                  up vote
                                  0
                                  down vote













                                  From here by standard limits



                                  $$ldots=lim_{xto 0} frac{a sin(ax) cos(bx)}{b sin(bx)cos(ax)}=lim_{xto 0} frac a bfrac{tan (ax)}{tan (bx)}=lim_{xto 0} frac {a^2} {b^2}frac{tan (ax)}{ax}frac{bx}{tan (bx)}=frac {a^2} {b^2}cdot 1 cdot 1 =frac {a^2} {b^2}$$






                                  share|cite|improve this answer

























                                    up vote
                                    0
                                    down vote













                                    From here by standard limits



                                    $$ldots=lim_{xto 0} frac{a sin(ax) cos(bx)}{b sin(bx)cos(ax)}=lim_{xto 0} frac a bfrac{tan (ax)}{tan (bx)}=lim_{xto 0} frac {a^2} {b^2}frac{tan (ax)}{ax}frac{bx}{tan (bx)}=frac {a^2} {b^2}cdot 1 cdot 1 =frac {a^2} {b^2}$$






                                    share|cite|improve this answer























                                      up vote
                                      0
                                      down vote










                                      up vote
                                      0
                                      down vote









                                      From here by standard limits



                                      $$ldots=lim_{xto 0} frac{a sin(ax) cos(bx)}{b sin(bx)cos(ax)}=lim_{xto 0} frac a bfrac{tan (ax)}{tan (bx)}=lim_{xto 0} frac {a^2} {b^2}frac{tan (ax)}{ax}frac{bx}{tan (bx)}=frac {a^2} {b^2}cdot 1 cdot 1 =frac {a^2} {b^2}$$






                                      share|cite|improve this answer












                                      From here by standard limits



                                      $$ldots=lim_{xto 0} frac{a sin(ax) cos(bx)}{b sin(bx)cos(ax)}=lim_{xto 0} frac a bfrac{tan (ax)}{tan (bx)}=lim_{xto 0} frac {a^2} {b^2}frac{tan (ax)}{ax}frac{bx}{tan (bx)}=frac {a^2} {b^2}cdot 1 cdot 1 =frac {a^2} {b^2}$$







                                      share|cite|improve this answer












                                      share|cite|improve this answer



                                      share|cite|improve this answer










                                      answered Nov 17 at 11:58









                                      gimusi

                                      89.7k74495




                                      89.7k74495






























                                          draft saved

                                          draft discarded




















































                                          Thanks for contributing an answer to Mathematics Stack Exchange!


                                          • Please be sure to answer the question. Provide details and share your research!

                                          But avoid



                                          • Asking for help, clarification, or responding to other answers.

                                          • Making statements based on opinion; back them up with references or personal experience.


                                          Use MathJax to format equations. MathJax reference.


                                          To learn more, see our tips on writing great answers.





                                          Some of your past answers have not been well-received, and you're in danger of being blocked from answering.


                                          Please pay close attention to the following guidance:


                                          • Please be sure to answer the question. Provide details and share your research!

                                          But avoid



                                          • Asking for help, clarification, or responding to other answers.

                                          • Making statements based on opinion; back them up with references or personal experience.


                                          To learn more, see our tips on writing great answers.




                                          draft saved


                                          draft discarded














                                          StackExchange.ready(
                                          function () {
                                          StackExchange.openid.initPostLogin('.new-post-login', 'https%3a%2f%2fmath.stackexchange.com%2fquestions%2f3002207%2fdo-not-understand-lhopital-rule%23new-answer', 'question_page');
                                          }
                                          );

                                          Post as a guest















                                          Required, but never shown





















































                                          Required, but never shown














                                          Required, but never shown












                                          Required, but never shown







                                          Required, but never shown

































                                          Required, but never shown














                                          Required, but never shown












                                          Required, but never shown







                                          Required, but never shown







                                          Popular posts from this blog

                                          Plaza Victoria

                                          In PowerPoint, is there a keyboard shortcut for bulleted / numbered list?

                                          How to put 3 figures in Latex with 2 figures side by side and 1 below these side by side images but in...